Jump to content

Lorenz Force thrust


DandelionTheory

Recommended Posts

What say you smarty pants?

I was way into the impossible em drive, smoked a huuuuge bowl and discovered the basic idea.

If the hypotenuse of a right triangle is open for charge carriers to be free, force applied to the system from the hypotenuse can be negated due to it not being physically attached. If 2 "circuits" are combined at the adjacent leg, the force on each of those legs is cancelled out in the system. What's left is force on the opposite legs.

-DT

Lol #winning

https://www.reddit.com/u/DandelionTheory?utm_medium=android_app&utm_source=share

20191108_123606.jpg

Edited by DandelionTheory
Specificity
Link to comment
Share on other sites

6 minutes ago, Strange said:
!

Moderator Note

Moved to Speculations. Please support your claims with evidence or mathematics.

 

FORCE ON A CURRENT CARRYING WIRE.

ITS CALLED THE LORENZ FORCE.

I honestly need to put it down? Can you not read or reference it yourself? I don't need to back up a WELL KNOWN CALCULATION STRANGE.

grow up.

Link to comment
Share on other sites

1 hour ago, DandelionTheory said:

smoked a huuuuge bowl

Reducing the smoking may allow for clearer drawings to be provided?

 

1 hour ago, DandelionTheory said:

What's left is force on the opposite legs.

What force is required to accelerate the "free ions"? What does the math say once that is taken into account?

 

Link to comment
Share on other sites

4 minutes ago, DandelionTheory said:

FORCE ON A CURRENT CARRYING WIRE.

ITS CALLED THE LORENZ FORCE.

I honestly need to put it down?

!

Moderator Note

If someone asked their teacher how an electric motor worked and the teacher replied "LORENTZ FORCE, DUMMY", the student would probably not find that very helpful.

Perhaps you could show exactly how the Lorentz force applies in the case of your diagram, and what the result is.

On the other hand, if you prefer not to follow the rules, we can just close this thread.

 
Link to comment
Share on other sites

8 minutes ago, Ghideon said:

Then just show the calculations that predict there is a net force on the rig in the drawing.

I did, with little blue F's

11 minutes ago, Strange said:
!

Moderator Note

If someone asked their teacher how an electric motor worked and the teacher replied "LORENTZ FORCE, DUMMY", the student would probably not find that very helpful.

Perhaps you could show exactly how the Lorentz force applies in the case of your diagram, and what the result is.

On the other hand, if you prefer not to follow the rules, we can just close this thread.

 

So how are ideas shared?

Is it with words and numbers? Can I use reference?

It's condescending to tell me to reference something when I referenced it. I would like some support communicating with you, but I'm having trouble. Do I really need to kiss you? Do I have to ask directly or do you just delete people that have trouble communicating?

Bro get up, get your big boy pants on and teach me.

Edited by DandelionTheory
Link to comment
Share on other sites

1 hour ago, DandelionTheory said:

If the hypotenuse of a right triangle is open for charge carriers to be free, force applied to the system from the hypotenuse can be negated due to it not being physically attached.

Do you know that when charged particle accelerates (i.e. changes direction of flight) it emits photons ? So electrons from your example will be emitting photons on each corner of your triangle and losing their kinetic energy.

https://en.wikipedia.org/wiki/Bremsstrahlung

https://en.wikipedia.org/wiki/Cyclotron_radiation

https://en.wikipedia.org/wiki/Synchrotron_radiation

 

Link to comment
Share on other sites

5 minutes ago, Sensei said:

Do you know that when charged particle accelerates (i.e. changes direction of flight) it emits photons ? So electrons from your example will be emitting photons on each corner of your triangle and losing their kinetic energy.

https://en.wikipedia.org/wiki/Bremsstrahlung

https://en.wikipedia.org/wiki/Cyclotron_radiation

https://en.wikipedia.org/wiki/Synchrotron_radiation

 

Man that would suck if they weren't in a circuit.

Yet.....

Maybe, maybe if it was in a vacuum.

Look at current direction.

20191108_151202.jpg

Link to comment
Share on other sites

17 minutes ago, DandelionTheory said:

I did, with little blue F's

Those little blue F's are just blue arrows. Not calculations.

How does the charged particles ("free ions") follow a straight line when there is a magnetic field?

Link to comment
Share on other sites

5 minutes ago, DandelionTheory said:

Maybe, maybe if it was in a vacuum.

CERN & LHC suffer from such radiation. And it happens in one of the best vacuum human can create.. Because it has nothing to do with collisions with atoms present in the air! Did you read links that I gave?

Link to comment
Share on other sites

Just now, Sensei said:

CERN & LHC suffer from such radiation. And it happens in one of the best vacuum human can create.. Because it has nothing to do with collisions with atoms present in the air! Did you read links that I gave?

You don't understand what I'm doing, first understand what I'm saying then you can maybe do something about correcting it.

What happens to each current-carrying wire?

the total force on each current-carrying wire is in the positive y direction.

3 minutes ago, Ghideon said:

Those little blue F's are just blue arrows. Not calculations.

How does the charged particles ("free ions") follow a straight line when there is a magnetic fiel

Some vector directions cancel out.

Link to comment
Share on other sites

2 minutes ago, DandelionTheory said:

You don't understand what I'm doing, first understand what I'm saying then you can maybe do something about correcting it.

!

Moderator Note

If you don’t start explaining yourself (instead of blaming others) and showing the calculations of the forces, then this thread will be closed. 

 
42 minutes ago, DandelionTheory said:

So how are ideas shared?

Is it with words and numbers? Can I use reference?

!

Moderator Note

Yes, some mathematics would be good. And more words. 

 
Link to comment
Share on other sites

4 minutes ago, DandelionTheory said:

the total force on each current-carrying wire is in the positive y direction.

Can you show a complete set of calculations, not just a formula? If you have a total force on current carrying wires then there is an opposite and equal force somewhere else in the rig or you are ejecting particles or radiations somewhere. Can you show calculations for conservation of momentum?   

Link to comment
Share on other sites

2 hours ago, Strange said:
!

Moderator Note

If you don’t start explaining yourself (instead of blaming others) and showing the calculations of the forces, then this thread will be closed. 

 
!

Moderator Note

Yes, some mathematics would be good. And more words. 

 
 
in the attached picture i attempt to show the force I'm working with.
The idea is to exploit the force on AB.
although the currents in the example DO NOT CREATE A CIRCUIT, they show the force im working with. 
IF, the circuit were to be closed with a wire from D to C, the  total force on DC would cancel out the total force on AB.
also,
We find the magnetic field vector direction by using the right hand rule. Because the current vector is inline with the writing surface, the magnetic field vector is represented as an X for into the surface and O for the vectors outward direction.
arguments.thumb.jpg.3adb066bffcb54b852952520f9b7324c.jpg
 
2 hours ago, Ghideon said:

Those little blue F's are just blue arrows. Not calculations.

How does the charged particles ("free ions") follow a straight line when there is a magnetic field?

Magnetic fields at 90 angles from an ions velocity vector do something curly to its path, but still follow electric field lines.

2 hours ago, Sensei said:

CERN & LHC suffer from such radiation. And it happens in one of the best vacuum human can create.. Because it has nothing to do with collisions with atoms present in the air! Did you read links that I gave?

not talking about radiation. im talking about magnetic fields acting on currents in wires and free ions. change your drugs bro.

Edited by DandelionTheory
Link to comment
Share on other sites

 

On 11/9/2019 at 2:43 AM, DandelionTheory said:

not talking about radiation. im talking about magnetic fields acting on currents in wires and free ions.

If you choose to neglect important concepts the result will be incorrect. For instance finding a net force on the rig when there is no such net force.

What happens to the current at points C, D and E? 

 

Link to comment
Share on other sites

  • 2 months later...
On 11/9/2019 at 2:43 AM, DandelionTheory said:

Magnetic fields at 90 angles from an ions velocity vector do something curly to its path, but still follow electric field lines.

Incorrect. A charged particle is affected by both the magnetic and the electric field. You need to take both fields into account. The resulting force acting on the particle does not necessarily accelerated the particle along a path that follows the electric field lines. If you claim otherwise, please provide the math so it can be reviewed.

Link to comment
Share on other sites

Do currents have to be attached to wires or is dielectric breakdown not a thing? You know if the electric field was in the 30kv range and the distance from E to C or D was 1 inch I'm sure some momentum would be lost in the discharge after the air breaks down. Wouldn't that be inline with conservation of momentum?

Or you know, resonant frequency isn't a thing either right? Maybe little capacitors on the butt end of each wire could resonate a charge. But whatever I'm done trying.

Edited by DandelionTheory
Link to comment
Share on other sites

1 hour ago, DandelionTheory said:

Do currents have to be attached to wires or is dielectric breakdown not a thing? You know if the electric field was in the 30kv range and the distance from E to C or D was 1 inch I'm sure some momentum would be lost in the discharge after the air breaks down. Wouldn't that be inline with conservation of momentum?

Or you know, resonant frequency isn't a thing either right? Maybe little capacitors on the butt end of each wire could resonate a charge. But whatever I'm done trying.

Please provide the math supporting your claims. 

 

(Trying to avoid issues by raising questions does not work)

Link to comment
Share on other sites

On 1/25/2020 at 1:02 PM, Ghideon said:

Incorrect. A charged particle is affected by both the magnetic and the electric field. You need to take both fields into account. The resulting force acting on the particle does not necessarily accelerated the particle along a path that follows the electric field lines. If you claim otherwise, please provide the math so it can be reviewed.

Cool, why? Show me some evidence why it's wrong please. It's condescending to demand clarification on an assumption of an assumption. I never said I claimed otherwise. Can you maybe get what I'm saying before it's assumed incorrect? 

Link to comment
Share on other sites

14 minutes ago, DandelionTheory said:

Cool, why? Show me some evidence why it's wrong please. It's condescending to demand clarification on an assumption of an assumption. I never said I claimed otherwise. Can you maybe get what I'm saying before it's assumed incorrect? 

!

Moderator Note

The burden is on you to provide information about your conjecture. people don't have to justify pointing out things that are incomplete, incoherent, or contrary to mainstream physics.

 
On 11/8/2019 at 8:43 PM, DandelionTheory said:
IF, the circuit were to be closed with a wire from D to C, the  total force on DC would cancel out the total force on AB.
also,

If the circuit isn't completed, there is no current. So you have no force. (also the current FE must be 2I, if you had a closed circuit)

I have no idea what your actual claim is. You need to explain it better. 

Link to comment
Share on other sites

41 minutes ago, swansont said:

If the circuit isn't completed, there is no current. So you have no force.

Okay I guess I have to complete each circuit before force arguments are even considered. I could set up a resonant circuit that would satisfy this current setup. I don't see how that's an impossibility, or do I need to specify the calculation is for a snapshot?

The first post claimed this chucked ions and the current in the wire experienced the lorentz force. It's not a tall order bro.

Edited by DandelionTheory
Link to comment
Share on other sites

4 minutes ago, DandelionTheory said:

Okay I guess I have to complete each circuit before force arguments are even considered. I could set up a resonant circuit that would satisfy this current setup. I don't see how that's an impossibility, or do I need to specify the calculation is for a snapshot?

If you are insisting there is a net force somewhere, then yes, we want to see a calculation. Your OP seems to claim there will be a net force, but later you admit that the force on different parts of the circuit will cancel. Which is why I said you need to explain things better. Your claim is unclear, which causes difficulty in figuring out what you are trying to explain.

Berating people for asking you to clarify makes things worse.

Link to comment
Share on other sites

Create an account or sign in to comment

You need to be a member in order to leave a comment

Create an account

Sign up for a new account in our community. It's easy!

Register a new account

Sign in

Already have an account? Sign in here.

Sign In Now
×
×
  • Create New...

Important Information

We have placed cookies on your device to help make this website better. You can adjust your cookie settings, otherwise we'll assume you're okay to continue.